CLP - Other Crimes

अब Quizwiz के साथ अपने होमवर्क और परीक्षाओं को एस करें!

An engineer, believing that an elderly woman suffered from arthritis, told her that for $100 he could cure her with a device he had invented. The device was a large box with a series of electric light bulbs along the sides. The woman, after examining the device, agreed to take the treatment, which consisted of placing her hands inside the box for several 10-minute periods. The woman gave the engineer $100 and went through the treatment. The engineer is charged with obtaining money by false pretenses. Which of the following, if true, would NOT absolve the engineer of guilt for obtaining money by false pretenses?

A conviction for obtaining property by false pretenses requires proof that the defendant: knowingly misrepresented a past or existing material fact did so with the specific intent to defraud (ie, no intent to return property) and thereby obtained title to the victim's property. There must be a causal connection between the defendant's misrepresentation and the transfer of property. Therefore, proof that the victim relied on the misrepresentation—whether reasonably or not—is also required. Here, the engineer's belief that the device would cure arthritis would negate the knowing misrepresentation requirement even if his belief was unreasonable (Choice A). And there would be no intent to defraud if the engineer was playing a practical joke on the woman and intended to return her money (Choice B). Additionally, a causal connection between the misrepresentation and property transfer would only exist if the woman believed that the device would cure arthritis (Choice D). Therefore, the woman's belief that the device would cure arthritis would support (not absolve) the engineer's guilt—regardless of whether her belief was reasonable.

In a crowded football stadium, a man saw a wallet fall out of a spectator's purse. The man picked up the wallet and found that it contained $100 in cash. Thinking that he could use the money and seeing no one watching, the man put the wallet in the pocket of his coat. Just then, the spectator approached the man and asked if he had seen a missing wallet. The man said no and went home with the wallet. Of what crime, if any, is the man guilty?

A person who finds lost or misplaced personal property is guilty of larceny if he/she: knows the identity of the owner or has reason to believe that he/she can determine it takes and carries away the property—ie, exercises control over and moves it and possesses the specific intent to permanently deprive the owner of the property. Here, the man saw the spectator's wallet fall out of her purse (knowledge of owner's identity). He picked up the wallet (taking and carrying away), noticed that it contained $100, and decided to keep it (specific intent to permanently deprive). Therefore, the man is guilty of larceny

A man who had become very drunk left a bar and started to walk home. Another patron of the bar, who had observed the man's condition, followed him. The patron saw the man stumble and fall to the ground near an alley. The patron then began to pull out a gun but saw that the man had passed out in the gutter. The patron reached into the man's pocket, grabbed his wallet, and started to walk away. When the patron heard police officers approaching, he dropped the wallet and ran off. The crimes below are listed in descending order of seriousness. What is the most serious crime of which the patron properly could be convicted?

A defendant can be convicted of larceny if he/she: unlawfully took and carried away the victim's property (even a short distance) with the intent to permanently deprive the victim of that property (ie, intent to steal). And the defendant can be convicted of the more serious crime of robbery if the property was taken from the victim's person or presence by force or intimidation. But if these (or any other) crimes are not completed, the defendant can be convicted of the less serious crime of attempt if he/she: specifically intended to commit the crime and performed an overt act in furtherance of the crime. Here, the elements of larceny were met when the patron intentionally took a wallet from the man's pocket and started to walk away. And though the patron later dropped the wallet when he heard the police approaching, this is no defense. Therefore, the patron can be convicted of larceny—not attempted larceny

One evening on a college campus, a student handed a classmate cash and the classmate gave the student a packet of heroin. The student looked at the packet and went to place it in his pants pocket. An off-duty police officer who was taking classes at the college witnessed the transaction from only a few feet away. The officer immediately identified herself as a police officer and told the participants that they were under arrest. The student, who had been in the act of placing the packet in his pants pocket, dropped the packet and unsuccessfully attempted to flee. The student was charged with possession of heroin. At trial, the student offered evidence, which the prosecution did not dispute, that the cash was not transferred in payment for the heroin but instead to repay a loan the classmate had made to the student. Can the student properly be convicted?

A defendant can be convicted of possession of a controlled substance (or other prohibited object) if the prosecution establishes that the defendant knowingly possessed the controlled substance. The possession element requires proof that the defendant either: knowingly received an illegal substance or exercised dominion and control over the substance after learning of its illegal character. Here, the student may not have initially known that the classmate had given the student a packet of heroin. However, the student had time to look at the packet and went to place it in his pants pocket before the police officer intervened. This shows that the student exercised dominion and control over the heroin. As a result, the student can be properly convicted.

Embezzlement is the: conversion of lawfully possessed property that belongs to another with the specific intent to defraud the person of that property. Conversion occurs when a person substantially interferes with another's rights to the property without consent (eg, unpermitted sale or destruction of another's property). Here, the pawnshop owner acquired a diamond ring that the jeweler used to secure a loan (lawful possession). They agreed that the jeweler had 60 days to repay the loan and that the pawnshop owner would not sell the ring before that time. Nevertheless, he prematurely sold the ring (conversion) since he thought the jeweler could not pay and could be handled if he did (intent to defraud). Therefore, the pawnshop owner committed embezzlement.

A defendant can be prosecuted in any state where an essential part of the crime has occurred without violating double jeopardy. Under the Model Penal Code, kidnapping occurs when a victim is intentionally and unlawfully moved (or confined) for any of the following purposes: to hold for ransom/reward or as a hostage/shield to facilitate commission of any felony or flight thereafter to inflict bodily injury on or terrorize the victim or another to interfere with performance of a governmental or political function An unlawful movement occurs when the victim is (1) removed from his/her residence or business or (2) taken a substantial distance from where he/she is found. And the movement must be more than incidental to the commission of another crime. To make this determination, courts consider the distance and duration of the movement as well as the increased danger the movement posed to the victim. Here, the stranger entered the car, pointed a gun at the driver, and unlawfully forced him to drive a substantial distance into a neighboring state. This was done so that the stranger could rob the driver of his wallet, cash, and car. And since the driver was (1) forced to drive several miles in the neighboring state and (2) at risk of being shot while doing so, his kidnapping was more than incidental to the robbery

In which of the following cases is a defendant most likely to be convicted if charged with receiving stolen property?

A defendant is guilty of receiving stolen property if he/she: took possession of, or exercised control over, stolen personal property knew that the property was stolen and specifically intended to permanently deprive the owner of the property. But property is no longer considered stolen once the police recover the property—ie, locate and exercise control over it. Here, the defendant bought a car from the man after the man told the defendant that it was stolen (knowledge and intent). The man stole the car with a friend who, unbeknownst to the man and the defendant, had been working as an undercover police informant. But since the police did not recover the stolen car before the defendant's purchase (possession), the defendant will likely be convicted of receiving stolen property.

A woman asked her neighbor if he would loan her $500, promising to repay the amount within two weeks. The neighbor loaned her the $500. The next day the woman took the money to the race track and lost all of it betting on horse races. She then left town for six months. She has not repaid the neighbor. Which of the following crimes has the woman committed?

A person who knowingly misrepresents a past or present material fact with the intent to defraud commits either: larceny by trick - if the person thereby obtains only possession of another's property or false pretenses - if the person thereby obtains title to another's property. Here, the woman did not commit larceny by trick since she obtained title to (not mere possession of) the neighbor's money when she failed to repay his loan as promised (Choices A & B). And she did not commit false pretenses since there is no indication that she knowingly misrepresented her ability to repay the loan or intended to defraud the neighbor (Choice C). Educational objective:False pretenses differs from larceny by trick in that false pretenses requires that a person obtain title to—not mere possession of—another's property.

A resident assistant visited a fellow college student in the student's dormitory room. They drank some beer. The student produced a box containing marijuana cigarettes and asked if the resident assistant wanted one. The resident assistant, afraid of being caught, declined and urged the student to get rid of the marijuana. The student refused. Shortly thereafter, both went out to get more beer, leaving the door to the student's room standing open. Making an excuse about having dropped his pen, the resident assistant went back into the student's room. Still apprehensive about their being caught with the marijuana cigarettes, the resident assistant took the cigarettes and flushed them down the toilet. He was sure the student was too drunk to notice that the cigarettes were missing. The jurisdiction defines burglary as the breaking and entering of the dwelling of another with intent to commit any felony or theft. If the resident assistant is charged with larceny and burglary, of which offense(s) can he properly be convicted?

A resident assistant visited a fellow college student in the student's dormitory room. They drank some beer. The student produced a box containing marijuana cigarettes and asked if the resident assistant wanted one. The resident assistant, afraid of being caught, declined and urged the student to get rid of the marijuana. The student refused. Shortly thereafter, both went out to get more beer, leaving the door to the student's room standing open. Making an excuse about having dropped his pen, the resident assistant went back into the student's room. Still apprehensive about their being caught with the marijuana cigarettes, the resident assistant took the cigarettes and flushed them down the toilet. He was sure the student was too drunk to notice that the cigarettes were missing. The jurisdiction defines burglary as the breaking and entering of the dwelling of another with intent to commit any felony or theft. If the resident assistant is charged with larceny and burglary, of which offense(s) can he properly be convicted?

Larceny v. Robbery

A robbery conviction requires proof that the defendant committed larceny—ie, an unlawful taking and carrying away of a victim's personal property with the intent to permanently deprive the victim of that property—plus two additional elements: that the property was taken from the victim's person or presence and that taking was accomplished by force or intimidation. The force requirement is met when the defendant uses greater force than necessary to take the victim's property. Therefore, a pickpocket does not commit robbery unless the victim resists the taking and the pickpocket then uses force to obtain or keep the property. Here, this defendant stole a wallet from the drunk man's vest without using force or intimidation. A minute later, the defendant slapped the man after being accused of taking the wallet. But that slap does not satisfy the force requirement because it was likely used to feign insult—not to forcibly keep the wallet. Therefore, compared to the other three cases, it is unlikely that this defendant's robbery conviction will be upheld. Robbery is a larceny that is (1) committed in the victim's presence and (2) achieved by force or intimidation. The force requirement is met when the defendant uses greater force than necessary to take the victim's property.

A skilled calligrapher crafted a letter on very old paper. She included details that would lead knowledgeable readers to believe the letter had been written by Thomas Jefferson to a friend. The calligrapher, who had a facsimile of Jefferson's autograph, made the signature and other writing on the letter resemble Jefferson's. She knew that the letter would attract the attention of local collectors. When it did and she was contacted about selling it, she said that it had come into her hands from a foreign collector who wished anonymity, and that she could make no promises about its authenticity. As she had hoped, a collector paid her $5,000 for the letter. Later the collector discovered the letter was not authentic, and handwriting analysis established that the calligrapher had written the letter. In a jurisdiction that follows the common law definition of forgery, which crime(s) has the calligrapher committed?

At common law (and in most jurisdictions), forgery occurs when a person: makes (ie, creates or alters) a false document of apparent legal significance with the intent to defraud. Documents of legal significance include checks, letters of recommendation, wills, diplomas, and other instruments that can effectuate one's intent to defraud. But such documents must have value beyond their own existence. Therefore, forgery cannot stem from instruments of mere social significance like historical documents (as seen here) and artwork (Choices A & B). However, a person who makes a false instrument of social significance can be liable for false pretenses if he/she: knowingly misrepresented a past or existing material fact did so with the specific intent to defraud and thereby obtained title to another's property or money. Here, the calligrapher intentionally crafted a letter that appeared to be written and signed by Thomas Jefferson. Although she made no representations regarding the letter's authenticity, she did misrepresent the letter's source by claiming that it had come into her hands from a foreign collector. And since that misrepresentation caused a local collector to buy the letter, the calligrapher committed the crime of false pretenses

A man revered a famous guitarist who had recently died. The man convinced his girlfriend, who worked at the cemetery where the guitarist was buried, to help him acquire a 12-foot sculpture of a steel guitar that had been commissioned by the guitarist's surviving spouse and installed on the guitarist's gravesite. One day, when the girlfriend knew that no other cemetery employees would be present, she gave the man the keys to a truck with a hydraulic lift that the cemetery used to install headstones. She and the man then drove to the guitarist's gravesite. The girlfriend maintained a lookout while the man severed the sculpture from its base and raised it onto the truck bed using the hydraulic lift. The man then drove the sculpture to a storage unit he had rented, placed it in the unit, and returned the truck to the cemetery. On the way home from the storage facility, the man was killed in a car accident. The police later ascertained the identities of the man and the girlfriend and recovered the sculpture unharmed. If the girlfriend is charged with larceny in a jurisdiction that follows the common law, which of the following is her best defense?

At common law, a person who (1) aided or encouraged a principal in the commission of a crime and (2) was present at the crime is classified as a principal in the second degree. This designation is significant because a principal in the second degree is liable for the offense to the same extent as the principal—even if the principal is not convicted. Here, the girlfriend aided the man in the unlawful removal of the sculpture from the guitarist's grave by serving as a lookout during the removal. As a result, the girlfriend is a principal in the second degree and can be tried and potentially convicted even though the man, who died before police could apprehend him, was not previously convicted (Choice A). For the girlfriend to be convicted of common law larceny, the prosecution must prove that the man: unlawfully took and carried away another's personal property and did so with the intent to permanently deprive the owner of that property. However, a person who unlawfully severs and then immediately carries away an item of real property—eg, a planted tree or fixture—is not guilty of common law larceny. That is because such an item does not constitute personal property in its attached state.* Therefore, the girlfriend's best defense is that the sculpture affixed to the grave constituted real property and, as such, was not subject to common law larceny. *In contrast, an item of real property that (1) was previously severed by the owner or (2) was severed by the defendant and later came into the owner's possession is personal property and subject to larceny.

A principal scientist recently left her job at a biotechnology corporation that manufactures and commercializes biopharmaceuticals for various medical conditions. The scientist began working for a competitor in order to lead the development of an experimental drug that would address the same medical condition as a drug she had been developing at the corporation. When the competitor issued a press release that it was in the final stages of testing the experimental drug, the corporation presented evidence to a prosecutor that the experimental drug contained the corporation's trade secrets and that the scientist first learned of the trade secrets while working at the corporation. The corporation also provided the prosecutor with a copy of its employment contract with the scientist, under which the scientist had agreed that the trade secrets belonged to the corporation and would remain with the corporation even after her employment ceased. In a common-law jurisdiction, which of the following property crimes is the prosecutor most likely to charge the scientist with?

At common law, liability for property crimes is limited to the trespassory or otherwise unlawful taking of tangible property.* Trade secrets are intangible property, so it is unlikely that the prosecutor will charge the scientist with a property crime in this common-law jurisdiction. Common-law property crimes limit criminal liability to the trespassory or otherwise unlawful taking of tangible property.

A man was short of money. He decided to go into a friend's house to take the friend's silverware and then to sell it. That night, while the friend was away, the man entered by picking the lock on the front door. He picked up a chest of silverware from the dining room and went out the front door of the house to his car. As he was putting the chest of silverware into the trunk, he had second thoughts and decided that he did not wish to become a thief. He reentered the house and replaced the chest of silverware where he had found it. As he came out of the house the second time, he was arrested by the police, who had been called by a neighbor. What is the most serious crime or crimes of which the man could be properly convicted?

Burglary is (1) the impermissible entry of a structure (2) with the intent to commit any crime therein. Larceny is (1) the unlawful taking and carrying away of another's personal property (2) with the intent to permanently deprive the owner of that property. Burglary is (1) the impermissible entry of a structure with (2) the intent to commit any crime therein. Larceny is (1) the unlawful taking and carrying away of another's personal property (2) with the intent to permanently deprive the owner thereof. And a change of heart after an offense has been completed does not negate criminal liability.

An executive of an accounting firm was fired and told to immediately leave the building where she worked. The executive went home, but she returned that night to retrieve personal items from her office. When she discovered that her key no longer opened a door to the building, she forced the door open and went to her former office. To avoid attracting attention, she did not turn on any lights. In the dark, she knew that she was taking some items that were not hers; she planned to sort these out later and return them. Upon arriving home, she found that she had taken a record book and some financial papers that belonged to the firm. After thinking it over and becoming angrier over being fired, she burned the book and papers in her fireplace. The jurisdiction has expanded the common law definition of burglary to include all buildings. What crime(s) has the executive committed?

Common law burglary is (1) the unlawful breaking and entering of another's dwelling at night (2) with the intent to commit a felony therein. Such intent must exist at the time of entry. And this jurisdiction (like most others) has expanded this definition to include all buildings—not just dwellings. Here, the executive was fired and told to immediately leave the building. When she returned to the building later that night, she discovered that her key no longer worked. She then forced the door open and went inside (unlawful breaking and entry). But since she entered the building with the intent to collect her personal items—not to commit a felony—the executive did not commit burglary. An executive of an accounting firm was fired and told to immediately leave the building where she worked. The executive went home, but she returned that night to retrieve personal items from her office. When she discovered that her key no longer opened a door to the building, she forced the door open and went to her former office. To avoid attracting attention, she did not turn on any lights. In the dark, she knew that she was taking some items that were not hers; she planned to sort these out later and return them. Upon arriving home, she found that she had taken a record book and some financial papers that belonged to the firm. After thinking it over and becoming angrier over being fired, she burned the book and papers in her fireplace. The jurisdiction has expanded the common law definition of burglary to include all buildings. What crime(s) has the executive committed?

A man was invited to attend a party at a neighbor's house. The man knew that the neighbor kept his valuable sports memorabilia collection in the basement game room and intended to steal it. On the night of the party, guests gathered on the main floor of the house and in the basement game room. The man made his way to the basement and waited until the number of guests dwindled to none. The man then went to grab the memorabilia but found that there was too much to carry out without being noticed. As a result, he decided to hide in the basement until the neighbor went to bed, at which time he would sneak out of the house with the memorabilia. Once the house was quiet, the man tried to exit the basement but found that the basement door had been closed and locked from the outside. Left with no other choice, the man broke a small basement window and climbed out of it with the memorabilia. The man has been charged with burglary in a common law jurisdiction

Common law burglary is the unlawful breaking and entering of another's dwelling at night with the specific intent to commit a felony therein. A breaking occurs when a person uses force to create an opening into a dwelling—eg, by opening an unlocked door or breaking a window to gain entry. But the use of force to exit a dwelling after commission of a felony therein does not constitute a breaking. Therefore, the man cannot be properly convicted of common law burglary (Choice C). The breaking element of common law burglary is satisfied when a person unlawfully uses force to create an opening into a dwelling. Therefore, use of force to exit a dwelling is insufficient to support this element.

A woman broke off her engagement to a man but refused to return the engagement ring the man had given her. One night, the man entered the woman's house after midnight to retrieve the ring. Although the woman was not at home, a neighbor saw the man enter the house and called the police. The man unsuccessfully searched for the ring for 10 minutes. As he was walking out the front door, the police arrived and immediately arrested him. The man has been charged with burglary in a jurisdiction that follows the common law. Which of the following, if proved, would serve as the man's best defense to the charge?

Common law burglary is the unlawful breaking and entering of another's dwelling at night with the specific intent to commit a felony therein. The intended felony is often larceny—ie: the unlawful taking and carrying away of another's personal property with the specific intent to permanently deprive the owner of that property. Therefore, a defendant's belief that he/she is entitled to the property—even if incorrect and unreasonable—serves as a defense to larceny since it negates the intent requirement (ie, mistake of fact defense). And if larceny is the intended felony for common law burglary, that belief is also a defense to burglary. Here, the man unlawfully entered the woman's house after midnight to retrieve the engagement ring he had given her. But he did not intend to commit a felony (ie, larceny) therein because he believed that he was legally entitled to that ring. Although this belief was incorrect and unreasonable, it still negates the intent requirement for larceny—and therefore burglary. As a result, this belief is the man's best defense.

Upon the recent passing of her mother, a sister inherited a valuable antique chair that her brother had coveted for a long time. Unbeknownst to the sister, the brother had accumulated a large amount of gambling debt that required immediate payment, so he decided to steal the chair when he knew that his sister would be out with friends. Early the next evening, just after the sister left, the brother entered her home through the front door, which he knew she always left unlocked. After entering the living room, the brother grabbed the chair and quickly exited the home. The sister's neighbor, who happened to be outside getting her mail, noticed the brother as he was leaving. She was able to see his face because it was illuminated by the setting sun. Upon recognizing him, she immediately called the police and reported the crime she had just witnessed. The brother was subsequently arrested and charged with burglary. In a common-law jurisdiction, is the brother guilty of burglary?

Common-law burglary is defined as the unlawful breaking and entering of another's dwelling at night with the specific intent to commit a felony therein—eg, larceny (as seen here). Nighttime is the period of darkness between sunset and sunrise. Therefore, it is not considered nighttime when there is sufficient natural daylight to see the burglar's face. Here, the brother entered the sister's home early in the evening while she was out with friends, with the intent to steal the sister's chair. But since it was early in the evening, there was sufficient sunlight for the neighbor to see the brother's face when he was outside. Therefore, the common-law nighttime requirement for burglary was not met.

A woman and her partner robbed a bank. Although they split the money they had stolen in half, the woman became greedy and decided to steal the partner's share. The woman knew that the partner had stashed his share of the money in a house that had been abandoned for some time. She decided to sneak into the house at night and steal the money. That night, the woman went to the back door of the house, and when she tried the doorknob, she discovered that it was unlocked. She snuck into the house, walked around until she found the partner's money in one of the rooms, and took it. A neighbor heard the woman rummaging in the house and immediately called the police. The police tracked the woman down and arrested her for burglary. In a jurisdiction that follows the common law with respect to criminal offenses, is the woman guilty of burglary?

Common-law burglary is the unlawful breaking and entering of another's dwelling at night with the specific intent to commit a felony therein. A dwelling is defined as a structure regularly occupied for habitation. Although the structure need not be occupied at the time of the breaking, it must not be abandoned. That is because common-law burglary seeks only to protect a person's right of habitation. Here, the woman broke into the house at night with the specific intent to steal the partner's money (ie, to commit larceny) (Choice C). However, the house was abandoned when she broke in. As a result, the house does not constitute a dwelling for purposes of a common-law burglary, and the woman is not guilty of this crime. (Choices A & D) A breaking can be accomplished by using even slight force, such as opening an unlocked door (as seen here). However, walking through an already open door would not constitute a breaking. Educational objective:For common-law burglary, a dwelling is defined as a structure regularly occupied for habitation. Although the structure need not be occupied at the time of the breaking, it must not be abandoned.

Two women, a boxer and a martial artist, agreed to compete against one another in a 12-round boxing match. Prior to the match, the referee cautioned both fighters that striking an opponent with an open glove, below the navel, in the kidneys, or behind the ears was not permitted. The referee further warned that hitting an opponent when she was knocked down on the ground was impermissible. Toward the end of the twelfth round, by which time the fighters had become exhausted, the boxer repeatedly punched the martial artist in the face with a closed glove. As the martial artist was falling to the ground, and before the referee could separate them, the boxer punched the martial artist in the face one more time and broke her nose. The martial artist subsequently filed a criminal complaint against the boxer for battery. If the boxer is indicted, what is her best defense against a charge of battery?

Criminal battery is the unlawful application of force to another that results in harmful or offensive contact. A victim's consent is generally not a defense to a crime. However, consent to bodily injury or conduct that may cause bodily injury is a defense when: the injury was not serious or the victim and the defendant were engaged in an activity in which the defendant's conduct and the victim's resulting injury were reasonably foreseeable. This is true regardless of whether the victim's consent was given explicitly (eg, by signed authorization) or implicitly (eg, by participating in an activity likely to result in injury). Here, both fighters consented to conduct that may cause bodily injury when they agreed to compete and participated in a boxing match. And since boxing is an activity in which the boxer's conduct (punch) and the martial artist's resulting injury (broken nose) were reasonably foreseeable, consent is the boxer's best defense against the charge of battery.

The ringmaster of a circus recognized a man sitting in the front row at one of his shows as his ex-wife's new boyfriend. Bitter and jealous, the ringmaster told one of the clowns who worked in the circus to choose the man to be a part of the show when it came time to involve members of the audience. He also instructed the clown to punch the man in the stomach once the other clowns were running around the stage and doing pranks for the audience. When it came time to involve audience members, the clown picked the man, and the man agreed to come on stage. The clown subsequently punched the man in the stomach as planned. The clown immediately regretted doing so, and he blurted out that the ringmaster had told him to do it. The man suffered internal bleeding as a result of the punch. Can the ringmaster be properly convicted of battery?

Criminal battery is the unlawful application of force to another that results in harmful or offensive contact. Although the force can be applied directly by the defendant, this requirement is also met if the defendant: ordered another person to apply force, thereby acting as an accomplice or set an object in motion that resulted in the application of force (eg, by throwing a rock at the victim). Here, the ringmaster ordered the clown to punch the man in the stomach (unlawful application of force). The clown did so, causing the man to bleed internally (harmful contact). As a result, the ringmaster can be properly convicted of battery. For battery, the defendant can unlawfully apply force to another by (1) applying the force directly, (2) ordering another person to apply force, or (3) setting an object in motion that results in the application of force.

Animal rights activists were protesting the use of monkeys in medical experiments by scientists who were developing a vaccine for a harmful virus. For several consecutive days, the activists marched in front of the exit door of the scientists' laboratory so that the scientists could not leave without first facing a barrage of name-calling and heckling from the activists. One scientist who was particularly agitated by the protesting decided to take revenge on one of the most vocal activists. On her way out of the laboratory, the scientist threw the skull of a monkey at the activist. The activist happened to turn his body and look away just after the scientist threw the skull. The skull struck the bag that the activist was carrying on his back. Upon discovering that he had been hit by a monkey skull, the activist became extremely distressed and upset. Is the scientist guilty of the crime of battery?

Criminal battery is the unlawful application of force to the victim's person—including an object near, carried by, or attached to the victim—that causes bodily harm or constitutes an offensive touching. Contact is offensive if a reasonable person would find it unpleasant or repugnant. Here, the scientist unlawfully threw a monkey skull (unlawful application of force) at the activist, striking the backpack that the activist was carrying (victim's person). Being struck by a monkey skull would be unpleasant or repugnant to a reasonable person, so the resulting contact was offensive. Therefore, the scientist is guilty of criminal battery

In which of the following situations is a defendant most likely to be guilty of the crime charged?

Defendant gets permission to borrow a car for the evening by falsely promising to return it. He has no intention of doing so. Two days later, the defendant changes his mind and returns the car. The defendant is charged with larceny by trick. A defendant is guilty of larceny by trick if he/she: obtained possession of another's personal property by knowingly making a false representation of a past or present material fact and had the specific intent to permanently deprive the owner of that property. A person cannot avoid criminal liability by returning the property after committing the crime. Here, the defendant received possession of another's car by falsely promising to return it when he intended to keep it. Although he later changed his mind and returned the car, he had already committed larceny by trick. Therefore, he will likely be convicted of this crime. Larceny by trick occurs when a person (1) obtains possession of another's personal property by knowingly making a false representation of a past or present material fact with (2) the specific intent to permanently deprive the owner of that property. And a person cannot avoid criminal liability by returning the property after committing the crime.

A designer properly placed a client's advance payment of anticipated fees into a client trust account. Several weeks later, the designer was asked by the state's Art and Design Association to serve as the chairperson of a committee overseeing an auction dinner to raise funds for the group. Lacking sufficient personal funds, the designer used money in the client trust account to pay for various expenses related to the dinner. The designer intended to fully repay the money in the trust account when the client abruptly fired the designer. Is the designer guilty of embezzlement?

Embezzlement is the fraudulent conversion of the property of another by a person who is in lawful possession of that property—typically pursuant to a trust agreement. Conversion is the inappropriate use of property that substantially interferes with the owner's right to the property. This occurs when the defendant treats the property as his/her own or uses it for his/her own purposes. Here, the designer was the trustee of the advance payment received from his client for anticipated services. The designer treated the money in the client trust account as his own and used it for his own purposes when he used it to pay for auction dinner expenses—not for fees incurred by the client. Therefore, the designer fraudulently converted the money and is guilty of embezzlement.

A jeweler took a diamond ring to a pawnshop and borrowed $200 on it. It was agreed that the loan was to be repaid within 60 days and if it was not, the pawnshop owner could sell the ring. A week before the 60 days expired, the pawnshop owner had an opportunity to sell the ring to a customer for $500. He did so, thinking it unlikely that the jeweler would repay the loan and if he did, the pawnshop owner would be able to handle him somehow, even by paying him for the ring if necessary. Two days later, the jeweler came in with the money to reclaim his ring. The pawnshop owner told him that it had been stolen when his shop was burglarized one night and that therefore he was not responsible for its loss. Larceny, embezzlement, and false pretenses are separate crimes in the jurisdiction. Which of the following crimes did the pawnshop owner commit?

Embezzlement is the: conversion of lawfully possessed property that belongs to another with the specific intent to defraud the person of that property. Conversion occurs when a person substantially interferes with another's rights to the property without consent (eg, unpermitted sale or destruction of another's property). Here, the pawnshop owner acquired a diamond ring that the jeweler used to secure a loan (lawful possession). They agreed that the jeweler had 60 days to repay the loan and that the pawnshop owner would not sell the ring before that time. Nevertheless, he prematurely sold the ring (conversion) since he thought the jeweler could not pay and could be handled if he did (intent to defraud). Therefore, the pawnshop owner committed embezzlement.

A man brought a valuable antique pocket watch to a pawnshop and borrowed $150 on it. The parties agreed that the man would repay the loan in 90 days and if he did not, the pawnshop owner could sell the watch. A week before the end of the 90-day period, a collector offered, and the pawnshop owner accepted, $400 in exchange for the watch. The pawnshop owner reasoned he would pay the man for the watch if he did come in. The man came in the next morning to pay $150 and retrieve the watch only to find that the pawnshop owner had sold it. When the man demanded his watch, the pawnshop owner said, "Look buddy, I'll give you the $400 I got for it and that's more than it's worth." Consequently, the man reluctantly took the money. Larceny, embezzlement, and false pretenses are separate crimes in the jurisdiction, of which offense could the pawnshop owner most appropriately be convicted?

Embezzlement is the: conversion of lawfully possessed property that belongs to another with the specific intent to defraud the person of that property. The intent to defraud exists when the defendant intends to permanently deprive the victim of the converted property. It is not negated if the defendant later (1) gives the victim cash for the converted property or (2) replaces it with similar property. Therefore, the pawnshop owner can be convicted of embezzlement even after giving the man $400 for the watch

A man owned an investment portfolio that, for the past several years, had consistently outperformed the stock market average. When the overall stock market precipitously declined in a single day, the man immediately contacted his financial advisor to ask about the status of his investment portfolio. The advisor lied to the man and told him that due to the decline in the stock market, the value of his account was $250,000, an amount significantly less than what the man had anticipated. However, the decreased value was not attributable to the decline in the stock market but rather the advisor's negligence in managing the portfolio. The man demanded that his account be closed and that the remaining balance in the account be transferred to him within the next two days. The advisor agreed and, accordingly, closed the man's account and transferred the remaining balance two days later. Unbeknownst to the man, there had been a temporary recovery in the investment portfolio that had resulted in a $5,000 increase in the man's account balance the day before his account was closed. The advisor retained the $5,000. In a jurisdiction that follows the common law with regard to property crimes, which crime has the advisor committed?

Embezzlement is: The conversion of lawfully possessed property that belongs to another with the specific intent to defraud the owner of that property. Conversion occurs when the person holding the property substantially interferes with the owner's right to the property—eg, by selling, damaging, or unreasonably withholding possession of the property. As a result, a conversion can be established by showing that the owner demanded the return of the property and the defendant failed to comply with that demand.* Here, the financial advisor lawfully possessed the man's account. The man requested the return of the balance, but the advisor failed to fully comply with that request because she retained $5,000. This substantially interfered with the man's right to those funds, and there is no indication that she intended to return them. Therefore, the advisor has committed embezzlement.

A man worked as the cashier in a restaurant. One night after the restaurant had closed, he discovered that the amount of cash in the cash register did not match the cash register receipt tapes. He took the cash and the tapes, put them in a bag, gave them to the manager of the restaurant, and reported the discrepancy. The manager immediately accused him of taking money from the register and threatened to fire him if he did not make up the difference. The manager placed the bag in the office safe. Angered by what he considered to be an unjust accusation, the man waited until the manager left the room and then reached into the still-open safe, took the bag containing the cash, and left. Of which crimes(s) is the man guilty?

Embezzlement occurs when a person: lawfully possesses another's property—ie, has the property owner's permission to possess or control it and fraudulently converts it—ie, substantially interferes with the owner's rights in property with the specific intent to defraud the owner of that property. Here, the man no longer lawfully possessed the restaurant's money once he turned it over to his manager. Therefore, even though he later took the money (substantial interference), he is not guilty of embezzlement (Choices A & C). Larceny is the (1) unlawful taking and carrying away of another's personal property (2) with the specific intent to permanently deprive the owner of that property. An unlawful taking occurs when the defendant removes property from the owner's possession and control without the owner's consent. And property is carried away once it is moved even a small distance.

An elderly woman was visiting a friend in a sizeable nursing home. After the woman's visit, she attempted to leave the facility. However, a nurse mistakenly believed that the woman was one of the residents and told her to remain on the premises. The woman protested and tried to explain that she was actually a visitor, but the nurse did not believe her. Instead, the nurse guided the woman toward an empty room and told her to wait there until he could verify her identity. The woman refused to enter the room, so the nurse pulled out a syringe and said, "Do I need to give you a sedative or are you going to listen to me?" The woman reluctantly entered the room, and the nurse told the woman to stay in there until he got back. He then closed the door behind him but did not lock it. Thirty minutes later, the nurse returned, apologized for the misunderstanding, and told the woman that she could leave. The woman subsequently filed a criminal complaint against the nurse, and he was charged with false imprisonment. Is the nurse guilty of false imprisonment?

False imprisonment is the unlawful confinement of a person without consent. A confinement occurs when a person is compelled through actual force, threat of force, or a show of force to either go where the person does not want to go or remain where the person does not want to remain. The confinement is unlawful unless it is specifically authorized by law or consented to by the victim. However, consent is only effective if it was freely given (ie, without force or fraud) and the victim had the capacity to consent. Here, the elderly woman was confined when the nurse threatened to give the woman a sedative unless she entered the room. Although the woman subsequently agreed to enter, her actions did not constitute consent because her compliance was not freely given—it was made in response to the nurse's threat of force. Therefore, the confinement was unlawful.

A man and a woman dated for several weeks. During that time, the man repeatedly asked the woman to have sex. Each time, the woman responded that she would not have sex with the man unless they were married. One evening, the man promised the woman that they would elope the following weekend if she would agree to have sex. The woman agreed and the couple had sex. The following weekend, the man told the woman that he had no intention of eloping and only made that promise to get the woman's consent. The woman reported the man to the police, who later arrested and charged the man with rape. Is the man guilty of rape?

In most modern jurisdictions, rape is defined as sexual intercourse with another without that person's consent.* This means that rape did not occur if the victim consented to sexual intercourse. However, a victim's consent may be ineffective if it was obtained by fraud. There are two types of fraud: Fraud in factum - when consent is obtained by fraud regarding the nature of the act itself, leaving the victim unaware that he/she consented to sexual intercourse and negating the victim's consent Fraud in the inducement - when consent is obtained by fraud regarding what the victim knows is an act of sexual intercourse, which does not negate the victim's consent As a result, consent obtained by fraud in factum is a defense to rape, but consent obtained by fraud in the inducement is not a defense. Here, the man falsely promised the woman that they would elope if she agreed to have sex with him. Since the woman knew that the act to which she consented was sexual intercourse, her consent was obtained by fraud in the inducement (Choices A & C). This type of fraud did not negate the woman's consent, so the man is not guilty of rape (Choice D). Fraud in factum occurs when the fraud pertains to the nature of the act itself and negates a rape victim's consent. In contrast, fraud in the inducement occurs when fraud is used to gain consent to what the victim knows is an act of sexual intercourse and does not negate the victim's consent

A woman decided to steal a necklace that belonged to her neighbor. She knew where the neighbor kept the necklace because she had been in the neighbor's house on many occasions when the neighbor had taken off the necklace and put it away in a jewelry box in the bathroom. One night, the woman went to the neighbor's house. The neighbor was away, and the house was dark. The woman opened the bathroom window, saw the jewelry box on the counter, and started to climb inside. As her leg cleared the window sill, the neighbor's dog began to bark loudly. Terrified, the woman jumped back outside and fled. The crimes below are listed in descending order of seriousness. What is the most serious crime, if any, committed by the woman?

In most jurisdictions, burglary is defined as the: unlawful entry of a building or other structure with the specific intent to commit any crime therein—even if that crime is not completed. An unlawful entry occurs when any part of the defendant's body (or an object under his/her control) crosses into the structure without the possessor's consent or legal privilege. And the crime that burglars often intend to commit inside is larceny—ie, the unlawful taking and carrying away of another's personal property with the specific intent to permanently deprive that person of the property.

A man and a woman went out on a date. At the end of the date, they went to the woman's apartment for a drink. As the night progressed, the man and the woman became amorous. The man suggested that they move into the woman's bedroom, and the two went into her dark bedroom and began to be sexually intimate. Unbeknownst to the man, the woman had a rare health condition that caused her to faint if her heart rate got too high. Prior to having intercourse, but before the woman gave her consent, the woman fainted. The man did not realize that the woman had fainted and, honestly believing that her lack of resistance indicated her consent, began having intercourse with her. A few moments later, the woman regained consciousness. When she realized what had happened, she reported the man to the police. The man was later arrested and charged with rape. Is the man guilty of rape?

In most modern jurisdictions, rape is sexual intercourse with another without that person's consent.* Rape is a general intent crime, which means that the defendant must consciously perform the unlawful act (Choice A). Mistake of fact is a defense to rape (and other general intent crimes) if the defendant's honest, reasonable, but mistaken belief negated the requisite mental state. For rape, this means that the mistake must have negated the intent to engage in nonconsensual sexual intercourse with the victim. Here, the woman fainted due to a health condition, leaving her unable to consent to sexual intercourse. However, the man honestly but mistakenly believed that the woman's lack of resistance indicated her consent. That mistake was likely reasonable because (1) the woman had voluntarily accompanied the man to the dark bedroom where they became intimate and (2) the man was unaware of the woman's health condition. Due to this mistake of fact, the man is not guilty of rape (Choices C & D). *At common law, rape was defined as (1) unlawful sexual intercourse (2) with a female who is not the defendant's wife (3) against her will by force or threat of force. Educational objective:Rape is a general intent crime. As a result, mistake of fact is a defense if the defendant's honest, reasonable, but mistaken belief negated the intent to engage in nonconsensual sexual intercourse with the victim.

During the night, a woman broke into a house with the intention of stealing a typewriter. On not finding a typewriter, she became angry, poured lighter fluid onto a couch, and set it on fire. The flames destroyed the couch and also burned a portion of the ceiling in the room. In a common law jurisdiction, of which crimes can the woman be found guilty?

In this common law jurisdiction, burglary is the (1) unlawful breaking and entering of another's dwelling at night (2) with the specific intent to commit afelony therein. The defendant must possess the requisite intent upon entering the dwelling but does not need to successfully complete the planned felony to be guilty of burglary. Therefore, the woman can be found guilty of burglary for breaking and entering another's home at night to steal a typewriter, even though she could not find and steal it (Choice A). Common law arson is the malicious (ie, intentional or reckless) burning of another's dwelling. Here, the woman acted maliciously when she set the couch on fire since she recklessly disregarded the high risk that the flames would spread to the rest of the house. And since the flames burned a portion of the ceiling, the woman can also be found guilty of arson (Choice D). (Choice C) Attempted arson occurs when the defendant (1) intends to burn another's dwelling, (2) commits an overt act in furtherance of that burning, but (3) does not complete the burning. Since the woman successfully burned part of the ceiling, she is guilty of arson—not attempted arson. Educational objective:Common law burglary occurs when the defendant unlawfully breaks and enters another's dwelling at night with the intent to commit a felony therein—even if the defendant failed to complete that felony. And common law arson occurs when the defendant maliciously burns another's dwelling.

A man entered the county museum at a time when it was open to the public, intending to steal an etching by a famous painter. Once inside, he took what he thought was the etching from an unlocked display case and concealed it under his coat. However, the etching was a photocopy of an original that had been loaned to another museum. A sign over the display case containing the photocopy said that similar photocopies were available free at the entrance. The man did not see the sign. Burglary in the jurisdiction is defined as "entering a building unlawfully with the intent to commit a crime." Of which crime(s) is the man guilty?

In this jurisdiction (and most others), burglary requires proof that a person unlawfully entered a building with the intent to commit a crime therein. Here, the man entered the museum with the intent to steal an etching. But since persons may lawfully enter a building held open to the public, his entry was lawful. Therefore, he is not guilty of burglary (Choices A & B). However, the man is guilty of larceny—ie, the unlawful (eg, nonconsensual) taking and carrying away of another's personal property with the specific intent to permanently deprive the owner thereof. The man took a photocopy of the etching from the display case even though other copies were available for free at the museum's entrance (unlawful taking). He then completed the crime when he concealed the photocopy under his coat (carrying away) with the intent to steal it (permanent deprivation). (Choices B & D) Attempt occurs when a person (1) has the specific intent to commit a crime, (2) engages in an overt act in furtherance of that crime, but (3) does not complete it. Since the man completed the larceny, he is not guilty of attempt. Educational objective:Burglary requires proof that a person unlawfully entered a building, so a person cannot burglarize a building while it is open to the public. And larceny requires proof that a person took another's personal property unlawfully—eg, without the owner's consent.

A woman told a man to accompany her into her friend's unlocked barn and retrieve an expensive black saddle that she said she had loaned to the friend. The man accompanied the woman to the friend's barn, opened the door, found a black saddle hanging high above the ground, and climbed a ladder to reach it. He handed the saddle down to the woman, and the two left with it together. In fact, the saddle belonged to the friend, and when the friend discovered the saddle missing, she suspected that the woman was the thief. The friend used a screwdriver to break into the woman's house to find the saddle. Upon discovering the saddle on the woman's kitchen table, the friend took it back and called the police. The jurisdiction follows the common law, except that burglary covers structures in addition to dwellings and the nighttime element has been eliminated. Which of these individuals is guilty of burglary?

In this jurisdiction, burglary is: the unlawful breaking and entering of another's structure or dwelling—eg, use of slight force to open a door and step through the doorway with the specific intent to commit a felony therein. Larceny is a felony that occurs when a person unlawfully takes and carries away another's personal property with the specific intent to permanently deprive the owner of it. And a person is guilty of these crimes if he/she personally committed the crime OR used an innocent agent to commit the criminal acts. An innocent agent is a person who lacked the requisite mens rea but was tricked or forced into committing the crime. A person is guilty if he/she personally commits a crime or uses an innocent agent to commit the criminal acts. An innocent agent is a person who lacked the requisite mens rea but was tricked or forced into committing the crime.

A man intended to rob a local convenience store that was rumored to contain a large safe. One night, the man walked into the convenience store, pointed a gun at the store owner, and demanded to know where the safe was kept. Frightened, the store owner pointed to a storage room located at the back of the store and approximately 20 feet from where the man and the store owner were standing. The man then moved the store owner to the storage room and forced him to open the safe. The man fled after taking the money in the safe, and the store owner immediately called the police. The man was later arrested and charged with robbery and kidnapping. Can the man be properly convicted of kidnapping?

Kidnapping is: the intentional and unlawful confinement of another against that person's will (eg, by force, threat, or fraud) coupled with either moving or hiding that person. A perpetrator need only move the victim a short distance—even a few feet—to satisfy the movement (ie, asportation) element of kidnapping (Choice B). However, for a kidnapping to occur incident to the commission of another offense, the movement of the victim must be more than is necessary to complete the other offense.* Otherwise, the perpetrator cannot be convicted of kidnapping. Here, the man entered the store intending to commit robbery. The man then pointed a gun at the store owner (force) and moved him into the storage room (confinement) so that the store owner could open the safe. But since the movement of the store owner was no more than was necessary to complete the robbery, the man cannot be properly convicted of kidnapping (Choices C & D). *Courts will consider several factors, including (1) the duration and significance of the movement, (2) whether the movement was an inherent part or independent of the other offense, and (3) whether the movement made the other offense easier to commit. Educational objective:For a kidnapping to occur incident to the commission of another offense, the victim must be moved more than is necessary to complete the other offense.

At a defendant's robbery trial, the state called the victim to testify that the defendant pointed a gun at him before grabbing his briefcase and running away. The state then called a bystander who testified that she saw the defendant take the victim's briefcase but saw no gun. For which of the following offense(s) could the jury properly return a verdict of guilty?

Larceny is (1) the unlawful taking and carrying away of another's personal property (2) with the specific intent to permanently deprive the victim of that property. It escalates to robbery when property is taken from the victim's person or presence by force or intimidation. As a result, larceny is necessarily committed during a robbery—making it a lesser included offense of robbery. And since larceny (lesser offense) merges into robbery (greater offense) once the robbery is completed, a defendant can only be convicted of larceny or robbery

In which of the following situations is a defendant most likely to be guilty of larceny?

Larceny is (1) the unlawful taking and carrying away of another's personal property with (2) the specific intent to permanently deprive that person of the property. Here, the defendant committed larceny when he took his ex-wife's dog and sold it. And though he believed that larceny does not include the taking of a dog, a mistake of law is almost never a defense.

A woman drove her car through the drive-through lane of a fast-food restaurant one afternoon. When she reached the microphone used to place orders, she said, "There's a man across the street with a rifle. He can see everything you do. If you do not do exactly what I tell you, he will shoot you. Put all the money from the register into a sack and give it to me when I drive up." The clerk did not see anyone across the street and was unsure whether anyone was there. However, unwilling to risk harm to himself, he put $500 in a paper bag and handed it to the woman when she drove up to the delivery window. The woman drove off with the money but was arrested a short time later. She had lied about there having been a man with a rifle and had acted alone. What is the most serious crime of which the woman can be convicted?

Larceny is (1) the unlawful taking and carrying away of the victim's personal property (2) with the specific intent to permanently deprive the victim of that property. And larceny becomes robbery when: the property is taken from the victim's person or presence and the taking is accomplished by force or intimidation. Since larceny is necessarily committed during robbery, it is a lesser included offense of robbery. And under the merger doctrine, a defendant can only be convicted of larceny (lesser offense) or robbery (greater offense). Here, the woman committed larceny when she took $500 from the clerk and drove away. But she acquired the money by threatening to have the clerk immediately shot if he did not comply. And since the clerk gave her the money because he was unwilling to risk being harmed, the woman can be convicted of the more serious crime of robbery Larceny becomes robbery when property is taken from the victim's person or presence by force or intimidation. Since larceny merges into the completed robbery, a defendant can be convicted of either crime—but not both.

While browsing in a clothing store, an actress decided to take a purse without paying for it. She placed the purse under her coat and took a couple of steps toward the exit. She then realized that a sensor tag on the purse would set off an alarm. She placed the purse near the counter from which she had removed it. Which crime, if any, has the actress committed?

Larceny is committed once a person (1) unlawfully takes and carries away another's personal property (2) with the specific intent to permanently deprive that person of his/her interest in the property. Carrying away (ie, asportation) occurs when property is moved even a short distance (eg, mere inches). And some jurisdictions only require that the property be brought under a person's dominion and control—no physical movement of the property is required. Here, the actress decided to take a purse without paying for it (specific intent). As a result, she removed the purse from the counter (unlawful taking) and concealed it under her coat (carrying away). Therefore, she committed larceny—even though the purse was never removed from the store. Larceny requires the carrying away (ie, asportation) of another's personal property. This occurs when the defendant moves the property a slight amount (majority rule) or brings the property under his/her dominion and control (minority rule).

A man who had retired from a career in music was asked to play some of his old songs at a local bar. Because he was anxious about singing after being retired for a few years, the man drank several shots of liquor at his friend's house and became intoxicated. Before he left for the bar, the man saw his friend's guitar, which was much nicer than the man's. The man wanted to give a good performance, so he took his friend's guitar to use that evening without the friend's permission. On his way to return the friend's guitar the next day, the man tripped and fell, breaking the friend's guitar outside of the friend's house. Upset by the man's actions, his friend called the police, and the man was charged with larceny. Can the man be properly convicted of larceny?

Larceny is the unlawful taking and carrying away of another's personal property (eg, guitar) with the specific intent to permanently deprive the owner of that property. Generally, the intent to permanently deprive the owner of the property must exist at the time of the taking. As a result, the requisite intent does not exist when, at the time of the taking, the defendant intends to: pay for merchandise that the defendant has the means to buy retrieve property that the defendant owns take money as repayment of a debt or borrow property with the ability to return it (as seen here). Under the "continuing trespass rule," larceny may be committed at a later time if a defendant who took property without the requisite intent later forms that intent. However, such an intent is not later formed merely because the property is accidentally damaged or destroyed. Here, the man did not intend to deprive the friend of his guitar indefinitely. Instead, he accidentally broke the guitar while he was on his way to return it to the friend (Choice D). As a result, the man cannot be properly convicted of larceny.

A woman who owned a prestigious art gallery was looking for paintings by a recently deceased artist. One of her clients had offered to pay over market price for one of the artist's paintings if the woman could deliver it to the client's home in time for a lavish party that he was throwing the following week. The woman was unable to acquire any of the artist's paintings. However, she knew that her grandfather, an avid art collector, owned one of the artist's earlier works. Knowing that her grandfather was out of town on vacation, the woman decided to have one of her employees pick up the painting from her grandfather's house. The woman falsely told the employee that the grandfather had given her the painting as a gift. When the employee arrived at the grandfather's house, he noticed that the garage door was open and saw the painting leaned up against a wall. The employee walked into the garage, took the painting, and delivered it to the woman. When the grandfather returned from his vacation the following day, he realized that the painting was missing and immediately called the police. Can the woman be properly convicted of larceny?

Larceny is the unlawful taking and carrying away of another's personal property with the specific intent to permanently deprive the owner of that property. The taking requirement is satisfied by any trespassory removal of the property from the owner's possession into another's control. As a result, the taking can be committed by the defendant personally or by the defendant's agent—even one who is unaware of the defendant's criminal intent. Here, the woman ordered her employee (agent) to take her grandfather's painting. Although the employee had no knowledge of the woman's criminal intent, an unlawful taking still occurred when he took the painting from the grandfather's garage and delivered it to the woman (Choice A). And since the woman had the specific intent to permanently deprive her grandfather of the painting at the time the taking occurred, she can be convicted of larceny.

An employee and her boss were leaving work late one evening. As they were passing a dark alleyway on the way to their cars, the defendant jumped out holding a knife. The defendant abruptly pushed the boss against the wall of a nearby building, held the knife to the boss's throat, and demanded the watch on his wrist. The boss complied. The defendant then turned to the employee and threatened to stab the boss if the employee did not hand over her money. The employee handed her wallet to the defendant, who took the wallet and ran away. Shortly thereafter, the police found and arrested the defendant. What crime(s) can the defendant be properly convicted of?

Larceny is the unlawful taking and carrying away of the victim's personal property with the specific intent to permanently deprive the victim of that property (ie, to steal). Larceny is elevated to robbery when: the property is taken from the victim's person or presence and the taking is achieved by force or threat of force against the victim, the victim's family member, or another person present. The threat-of-force element of robbery can be satisfied by a battery, which is the unlawful application of force to another resulting in bodily injury or an offensive touching. This element can also be satisfied by an assault, which is an attempted battery or conduct that intentionally puts the victim in reasonable apprehension of imminent bodily harm. Here, the defendant battered the boss by grabbing him, pushing him against a wall, and holding a knife to his throat. The defendant also assaulted the boss by threatening to stab him. But the defendant did not assault the employee since he did not attempt to batter her or place her in apprehension of bodily harm

Driving down a dark road, the defendant accidentally ran over a man. The defendant stopped and found that the victim was dead. The defendant, fearing that he might be held responsible, took the victim's wallet, which contained a substantial amount of money. He removed the identification papers and put the wallet and money back into the victim's pocket. Of which crime is the defendant NOT guilty?

Larceny is the unlawful taking and carrying away of the victim's property with the intent to permanently deprive the victim of that property (ie, intent to steal). And larceny is elevated to robbery when two additional elements are present: the property is taken from the victim's person or presence and the taking is accomplished through force or intimidation. Here, the defendant is guilty of larceny since he intentionally stole identification papers from the victim's wallet. It is not a defense that the victim was already dead since this does not mean that the defendant acquired a legal right to take the papers (Choice A). Nor is it a defense that the defendant took the papers to prevent identification (not for his own use) since that does not negate his intent to steal (Choice B). However, the defendant is not guilty of the more serious crime of robbery since he did not take the papers from the victim's person by means of force or putting in fear (the victim was already dead). But had the defendant done so, the fact that the papers had no monetary value would not excuse his liability for this crime (Choice C). Educational objective:Larceny is the unlawful taking and carrying away of the victim's property with the intent to permanently deprive the victim of that property. And larceny becomes robbery when the property is taken (1) from the victim's person or presence (2) through force or intimidation.

A husband and wife were walking to their car one evening after having seen a movie. As they were passing a dark alleyway, a youth leaped out brandishing a gun. He pushed the wife against the wall of a nearby building, held the gun to her head, and demanded money from the husband. The husband handed over his cash. The youth grabbed the cash and ran away. Which of the following, listed in descending order of seriousness, is the most serious crime of which the youth may be convicted?

Larceny is the unlawful taking and carrying away of the victim's property with the intent to permanently deprive the victim of that property (ie, to steal). And larceny is elevated to robbery when: the property is taken from the victim's person or presence and the taking is achieved by force or threat of force against the victim, the victim's family member, or another person present. The threat-of-force element of robbery can be satisfied by an assault—conduct that intentionally puts another person in reasonable apprehension of imminent bodily harm. Here, the youth committed assault on both the husband and the wife (by brandishing a gun at them) and the wife again (by holding a gun to her head). The youth then committed the more serious crime of larceny when he demanded money from the husband and took the cash from him. And since the youth accomplished that taking by means of assault, he can be convicted of robbery—the most serious crime listed

A man admired his coworker's wristwatch and frequently said how much he wished he had one like it. The coworker decided to give the man the watch for his birthday the following week. On the weekend before the man's birthday, the man and the coworker attended a company picnic. The coworker took his watch off and left it on a blanket when he went off to join in a touch football game. The man strolled by, saw the watch on the blanket, and decided to steal it. He bent over and picked up the watch. Before he could pocket it, however, the coworker returned. When the coworker saw the man holding the watch, the coworker said, "I know how much you like that watch. I was planning to give it to you for your birthday. Go ahead and take it now." The man kept the watch. Which crime has the man committed?

Larceny is the: unlawful taking and carrying away of another's personal property with the specific intent to permanently deprive the owner of that property (ie, intent to steal). An unlawful taking occurs when the defendant removes the property from the owner's possession and control without consent. Property is carried away as soon as the defendant moves it a short distance (eg, mere inches). And once the defendant has committed larceny, it is no defense that the owner later consented to the taking. Here, the man grabbed the coworker's watch without his permission (unlawful taking) and picked it up (carrying away) with the intent to steal it. Although the coworker then told the man that he could have the watch, the coworker's consent is not a defense because it was given after the man had committed larceny Larceny is the (1) unlawful taking and carrying away of another's personal property (2) with the specific intent to permanently deprive the owner of that property. And the owner's consent to the taking after the defendant has completed the larceny does not negate the defendant's criminal liability.

A customer asked to see an expensive watch in a jewelry store. In conversation with the store clerk, the customer falsely claimed to be the son of the mayor. When handed the watch, the customer asked if he could put it on, walk around a bit so he could see how it felt on his wrist, and briefly step outside to observe it in natural light. The clerk agreed, saying, "I know I can trust someone like you with the merchandise." The customer walked out of the store wearing the watch and did not return. A week later, the clerk was at a gathering when she spotted the customer wearing the watch. She told him that he must either pay for the watch or give it back. He hissed, "You'll be sorry if you mess with me." Intimidated, the clerk backed off. The following list of crimes is in descending order of seriousness. What is the most serious crime the customer committed?

Larceny occurs when a person (1) unlawfully takes and carries away another's personal property (2) with the specific intent to permanently deprive the owner of that property. An unlawful taking occurs when a person obtains possession of another's property without privilege or valid consent. This can arise when consent is obtained by knowingly misrepresenting a past or present material fact (ie, larceny by trick). Here, the customer took the watch and carried it outside with the intent to permanently keep it. And though the clerk gave the customer permission to take the watch, she only did so because the customer falsely told her that he was the mayor's son. As a result, the clerk's consent was invalid, and the customer committed larceny.

A man was angry at a neighbor with whom he had quarreled and, for revenge, surreptitiously removed a piece of stone statuary from the neighbor's garden and concealed it in his garage. The man intended to return the statue a day or two later, after giving the neighbor a chance to feel bad over its being stolen. Suspecting who was guilty, the neighbor had the defendant arrested and charged with larceny. Which case would be most applicable as a precedent?

Larceny requires proof of: an unlawful taking and carrying away of another's personal property (actus reus) with the specific intent to permanently deprive the person of that property (mens rea). Here, the man unlawfully took and concealed his neighbor's statue with the intent to return it, but he was arrested before he could do so. Therefore, the issue presented in this case is whether the man's intent to return the property negates the requisite mens rea for larceny—ie, the intent to permanently deprive—if he ultimately was unable to return it. And any applicable precedent should address a similar issue. In Saferite v. State, two men were convicted of larceny for unlawfully borrowing another's car to visit a friend. Although they intended to return the car, they could not do so since the car was wrecked on their way back. These convictions were reversed after the appellate court resolved a similar issue—holding that an intent to return property does negate the mens rea required for larceny even if the defendant was later unable to do so. Therefore, this case provides the most applicable precedent.

A chemistry student manufactured and sold methamphetamine to pay for her college tuition. One day, she mentioned to a friend that state law strictly limits purchases of certain medications used in the production of methamphetamine. Because the student could only purchase a small quantity of these medications at a time, she also mentioned that it would take her until the next semester to acquire enough ingredients to produce her next batch. That night, the friend asked the student for a ride to a local pharmacy to pick up a few items. Unbeknownst to the student, the friend stole several boxes of the medication that the student needed to make methamphetamine from the pharmacy shelf. The student then drove the friend back to the friend's dorm room. Later that week, the friend delivered the stolen boxes of medication to the student. When the student asked how the friend had gotten the medication, the friend admitted that she had stolen it from the pharmacy. The student offered to pay the friend for the medication, but the friend said, "Don't worry about it. Friends support each other." The student accepted the medication and started preparing a batch. On these facts, which of the following crimes has the student committed?

Receiving stolen property is a statutory crime that is committed when a person: receives control of stolen personal property knows that the property was stolen (eg, unlawfully obtained through larceny, embezzlement, false pretenses) and specifically intends to permanently deprive the owner of that property. Here, the student accepted the medication (received control) after the friend admitted that she had stolen it from the pharmacy (knowledge). The student then began preparing a batch of methamphetamine with the stolen medication, indicating her intention to permanently deprive the owner of the medication. Therefore, the student committed the crime of receiving stolen property. Receiving stolen property is committed when a person (1) receives control of stolen personal property, (2) knows that the property was stolen, and (3) specifically intends to permanently deprive the owner of that property.

A man and a woman planned to hold up a bank. They drove to the bank in the woman's car. The woman entered the bank while the man remained as lookout in the car. After a few moments, the man panicked and drove off. In the bank, the woman looked over the various tellers, approached one and whispered nervously, "Just hand over the cash. Don't look around, don't make a false move—or it's your life." The teller looked at the fidgeting woman, laughed, flipped her a dollar bill and said, "Go on, beat it." Flustered, the woman grabbed the dollar and left. Soon after leaving the scene, the man was stopped by the police for speeding. Noting his nervous condition, the police asked the man if they might search the car. The man agreed. The search turned up heroin concealed in the lid of the trunk. The man is charged with robbery. What is his best defense?

Robbery is the: unlawful taking and carrying away of a victim's personal property from the victim's person or presence by force or intimidation with the intent to permanently deprive the victim of that property. The intimidation requirement is met if the victim reasonably apprehends that he/she, a close family member, or another person present will be subjected to imminent bodily harm if the victim fails to surrender the property. Here, the man and woman intended to rob a bank. The man stayed in the car as the lookout while the woman entered the bank. When she whispered nervously to the bank teller, "Just hand over the cash...or it's your life," the teller laughed and flipped her a dollar bill (unlawful taking). The woman then left with the dollar (carrying away). But since the teller was not placed in fear of imminent bodily harm (no intimidation), no robbery occurred. An accomplice is someone who intentionally aids or encourages the principal to commit a crime with the intent that the principal complete the crime. Therefore, an accomplice is liable to the same extent as the principal. Here, although the woman entered the bank alone, the man acted as her accomplice when he helped plan the robbery and served as a lookout (Choice A). But he is not guilty of robbery since she did not complete the crime.

After waiting until all the customers had left, a man entered a small grocery store just before closing time. He went up to the lone clerk in the store and said, "Hand over all the money in the cash register or you will get hurt." The clerk fainted and struck his head on the edge of the counter. As the man went behind the counter to open the cash register, two customers entered the store. The man ran out before he was able to open the register drawer. Of which crime(s) could the man be properly convicted based on this evidence?

Robbery requires proof of all of the following elements: The defendant unlawfully took and carried away the victim's personal property. The property was taken from the victim's person or presence by force or intimidation (eg, assault). The defendant specifically intended to permanently deprive the victim of the property (ie, to steal). But a defendant who does not complete all of these elements can still be convicted of attempted robbery if he/she (1) had the specific intent to commit robbery and (2) committed an overt act in furtherance of that crime. Here, the man did not commit robbery since he ran out of the store before he could take the money (Choices B & D). But since he specifically intended to steal the cash by intimidation and committed an overt act in furtherance of the robbery when he threatened to hurt the clerk, he can be convicted of attempted robbery. Assault is (1) an attempted batteryor (2) conduct that intentionally puts the victim in reasonable apprehension of imminent bodily harm. Therefore, the man committed assault in the course of the attempted robbery when he threatened to hurt the clerk if he did not comply. But since assault satisfies the intimidation requirement for robbery, it is a lesser included offense. As a result, the assault merges with the attempted or completed robbery. This means that the man can be convicted of assault or attempted robbery—not both (Choice A). Educational objective:Robbery requires a taking of the victim's property by force or intimidation. Since assault satisfies that intimidation requirement, it is a lesser included offense that merges with the completed robbery or attempted robbery.

A man walked into a store that had a check-cashing service and tried to cash a $550 check that was payable to him. The attendant on duty refused to cash the check because the man did not have two forms of identification, which the store's policies required. The man, who had no money except for the check and who needed cash to pay for food and a place to sleep, became agitated. He put his hand into his pocket and growled, "Give me the money or I'll start shooting." The attendant, who knew the man as a neighborhood character, did not believe that he was violent or had a gun. However, because the attendant felt sorry for the man, he handed over the cash. The man left the check on the counter and departed. The attendant picked up the check and found that the man had failed to endorse it. Of which crime is the man guilty?

Robbery requires proof of all the following elements: The defendant unlawfully took and carried away the victim's personal property. The property was taken from the victim's person or presence by force or intimidation. The defendant specifically intended to permanently deprive the victim of the property (ie, to steal). But a defendant who does not complete all of these elements can still be convicted of attempted robbery if he/she (1) had the specific intent to commit robbery and (2) committed an overt act in furtherance of that crime. Here, the man intentionally took $550 in cash after he threatened the attendant by stating, "Give me the money or I'll start shooting." But this taking was not accomplished by force. Nor was it accomplished by intimidation since the attendant gave the man the money out of pity—not fear. As a result, the man is not guilty of robbery (Choice A). But since he specifically intended to steal the cash by intimidation and committed an overt act by threatening to shoot the attendant, the man is guilty of attempted robbery. Robbery requires proof that property was taken from the victim's person or presence by intimidation (or force). So if the victim was not fearful of the defendant's threat, any taking was not by intimidation.

Early one morning, a husband and wife were walking through an empty parking garage toward their sports car. As they neared the car, a man carrying a crowbar jumped out from behind the car and yelled, "Give me all of your jewelry and your money, or else I will beat your car to a pulp!" The husband and wife handed their jewelry and money to the man, and the man quickly departed. The man was later found and arrested for robbery. Is the man likely guilty of robbery?

Robbery requires proof of three elements: The defendant committed larceny—ie, a trespassory taking and carrying away of a victim's personal property with the specific intent to steal. The property was taken from the victim's person or in the victim's presence. The taking was accomplished by force or intimidation. A taking by intimidation occurs when there is an immediate threat of death or serious physical injury to the victim, a close family member, or another person present. But a threat to damage or destroy property (as seen here), other than the victim's home, is not sufficient. Therefore, the man likely is not guilty of robbery. For robbery, a taking by intimidation occurs when there is an immediate threat of death or serious physical injury to the victim, a close family member, or another person present. But a threat to damage or destroy property, other than the victim's home, is not sufficient.

A college student wanted to purchase a particular narcotic. She approached someone whom she believed to be a drug dealer about buying the narcotic. Unbeknownst to the college student, the person she approached was actually an undercover narcotics officer. The college student asked the undercover officer to sell her a small amount of the narcotic. Thereafter, the undercover officer immediately arrested the college student. A law in the jurisdiction criminalizes the sale of any amount of the narcotic. The college student was charged with solicitation. If the charges against the student are dropped, what is the most likely reason?

Solicitation occurs when a person entices, encourages, requests, or commands another to commit a crime with the specific intent that the solicited crime be committed. However, a person who does so (ie, the soliciting party) cannot be convicted of solicitation if: the solicited crime requires more than one participant (eg, the sale of a narcotic requires a buyer and a seller) the criminal statute only imposes liability on one participant (eg, only the seller can be convicted) and the soliciting party is the type of person the law was enacted to protect (eg, the buyer). The reason is that such persons would not be guilty as an accomplice if the solicited crime was completed because their conduct, though essential to commission of the crime, is not specifically prohibited by the statute. Therefore, the college student cannot be convicted of solicitation.* *In contrast, the college student would have been guilty of solicitation had she requested that the officer sell the narcotic to a third party because the student would no longer be acting as a buyer.

A 22-year-old man went to a nightclub with a group of friends. The nightclub is known for its strict 18-and-up policy. While there, the man ran into a former coworker who introduced the man to her sister. Over the course of the night, the man and the sister danced and engaged in conversation. The sister told the man that she was 19 years old and attended the local university. Near closing time, the sister invited the man to her apartment. Unbeknownst to him, the sister was only 16 and had gotten into the club using a fake ID. Later that evening, the man had consensual sexual intercourse with the coworker's sister. The age of consent in the jurisdiction is 17. Is the man guilty of statutory rape?

Statutory rape arises when a defendant has sexual intercourse with a person under the age of consent. This is a strict liability crime, which means that it has no mens rea requirement. Instead, proof of the actus reus (here, intercourse with an underage person) is alone sufficient to support a conviction. As a result, the fact that the man did not knowingly engage in sexual intercourse with a minor does not absolve him of guilt

The CEO of a company, who spent many of her work hours surfing stamp-collector websites, frequently ordered expensive stamps that she had delivered to her work address. One day, a very rare and expensive stamp arrived in the mail. The CEO's assistant, who was resentful about the meager pay he received from the CEO, decided to steal the stamp. He checked the CEO's calendar and saw that she was attending the opera the following evening. The next night, after the CEO left for the opera, the assistant retrieved a spare key hidden under a rock and entered the CEO's home through the front door, which led straight into the living room where the stamp collection was prominently displayed. As the assistant searched through the stamp collection for the rare stamp, the front door suddenly opened and the CEO walked in. Upon seeing the assistant, the CEO reached into her purse, pulled out a gun, told the assistant to stay where he was, and called the police. In a common law jurisdiction, which of the following crime(s) can the assistant be found guilty of?

The CEO of a company, who spent many of her work hours surfing stamp-collector websites, frequently ordered expensive stamps that she had delivered to her work address. One day, a very rare and expensive stamp arrived in the mail. The CEO's assistant, who was resentful about the meager pay he received from the CEO, decided to steal the stamp. He checked the CEO's calendar and saw that she was attending the opera the following evening. The next night, after the CEO left for the opera, the assistant retrieved a spare key hidden under a rock and entered the CEO's home through the front door, which led straight into the living room where the stamp collection was prominently displayed. As the assistant searched through the stamp collection for the rare stamp, the front door suddenly opened and the CEO walked in. Upon seeing the assistant, the CEO reached into her purse, pulled out a gun, told the assistant to stay where he was, and called the police. In a common law jurisdiction, which of the following crime(s) can the assistant be found guilty of?

A new lawyer has three clients, all of whom are indigent. To improve the appearance of his office, he decided to purchase some new furniture and to pay for it out of future earnings. Wearing an expensive suit borrowed from a friend, the lawyer went to a furniture store and asked to purchase on credit a desk and various other items of furniture. The lawyer told the store owner that he was a very able lawyer with a growing practice and that he expected to do very well in the future. The store owner agreed to sell him the items on credit, and the lawyer promised to make monthly payments of $800. The lawyer has never had an income from his practice of more than $150 a month. The lawyer's business did not improve, and he did not make any payments to the furniture store. After three months, the store owner repossessed the items. If the lawyer is charged with obtaining property by false pretenses, what is his best argument for being NOT guilty?

The crime of false pretenses requires proof that the defendant: knowingly misrepresented a past or existing material fact did so with the specific intent to defraud and thereby obtained title to the property of another. A material fact is a fact that a person would reasonably consider in deciding whether to transfer the property. Therefore, a misrepresentation cannot stem from an opinion or a prediction about a future event. Here, the lawyer told the store owner that he was a very capable attorney (opinion) with a growing practice and expected to do very well in the future (future event). And the store owner may have considered this statement when he agreed to sell the furniture to the lawyer on credit. But if the lawyer successfully argues that his statements were not misrepresentations of material fact, he would not be found guilty of false pretenses.

A woman went to an art gallery and falsely represented that she was an agent for a museum and wanted to purchase a painting that was hanging in the gallery. The woman and the gallery owner then agreed on a price for the painting to be paid 10 days later, and the woman took the painting. When the gallery failed to receive the payment when due, the owner called the museum and discovered that the woman did not work there. The owner then notified the police. When interviewed by the police, the woman admitted to making the false representation and acquiring the painting, but she said she believed that the painting had been stolen from her by someone who worked in the gallery. Is the woman guilty of obtaining property by false pretenses?

The crime of obtaining property by false pretenses occurs when a person: knowingly misrepresents a past or existing material fact does so with the specific intent to defraud and thereby obtains title to the property of another. Intent to defraud is the intent to induce the owner of the property to permanently part with it. Therefore, such intent does not exist if the defendant either (1) believes that he/she has the right to appropriate the property or (2) intends to and is able to return the property. Here, the woman falsely represented to the gallery owner that she was an agent for a museum and wanted to purchase a painting (knowing misrepresentation). They then agreed on a price to be paid 10 days later and the woman took the painting (obtained title). The woman never made that payment. But since she believed that the painting had been stolen by someone at the gallery and that it belonged to her (right to appropriate), she lacked the intent to defraud needed to be guilty of false pretenses. The crime of false pretenses requires proof that the defendant intended to defraud the property owner. Such intent does not exist when the defendant (1) believes that he/she has the right to appropriate the property or (2) intends to and is able to return it.

A woman and her coworker were invited to go on a hunting trip by their boss. The woman and the coworker were vying for the same promotion, so they were eager to impress their boss with their hunting skills. At one point while the woman and the coworker were hunting alone, the coworker pointed a gun at the woman while her back was turned and said, "Promise you will turn down the promotion or I will shoot you!" Unbeknownst to the coworker, the woman could not hear him because she had placed protective earplugs in her ears before the hunt began. The coworker believed that the woman was intentionally ignoring him, so he aimed at and killed a nearby deer, frightening the woman with the blast from the gun. If the coworker is charged with assault, is he likely to be convicted?

There are two types of criminal assault: Attempted battery - when the defendant has the specific intent to commit and takes a substantial step toward committing a battery "Fear of harm" assault (ie, apprehension assault) - when the defendant intentionally places another in reasonable apprehension of imminent harmful or offensive contact For "fear of harm" assault, the victim must have been aware of the defendant's threat of harm at the time it occurred. Because actual apprehension is necessary, the victim's lack of awareness of the threat of harm is a defense to this type of assault. Here, the coworker did not intend to batter the woman and took no substantial step toward doing so. He did, however, intend to place the woman in reasonable apprehension of imminent harmful contact (Choice A). But the woman was not aware of the coworker's threat of harm since her back was turned and she could not hear his threat due to her protective earplugs. Therefore, the coworker is not likely to be convicted of assault. (Choice C) Although the coworker had the requisite intent to cause apprehension of bodily harm, he did not cause the reasonable apprehension of such harm. Both elements must be met for the coworker to be convicted of "fear of harm" assault.

An unemployed man was planning to rob a convenience store and staked out the store for several weeks. One evening, the man opened and entered through the front door of the store just before closing time and reached for the gun in his waistband as he approached the checkout counter. However, upon seeing the elderly clerk who reminded the man of his grandfather, the man changed his mind and ran out of the store. Finding the man's behavior suspicious, the clerk called the police, who later apprehended the man and charged him with burglary. In the jurisdiction, burglary is defined as "breaking and entering any building or structure with the intent to commit a felony or to steal therein." Is the man likely to be found guilty of burglary?

This jurisdiction defines burglary as: the breaking and entering of any building or structure with the intent to commit a felony (eg, robbery) or to steal therein. Breaking occurs when the defendant uses any degree of force to gain entry (eg, opening a door), and entering occurs when any part of the defendant's body or an object in his/her possession crosses into the structure. The breaking and entering also must have been unlawful—ie, without the owner's permission or legal privilege. A business gives the public permission to enter its premises during business hours, but not when it is closed. Here, the man opened the front door and entered the convenience store with the intent to commit a robbery therein (Choice C). But since the store was still open to the public at that time, the man entered the store with the owner's permission. As a result, the man's entry was lawful, and he likely will not be found guilty of burglary. Burglary requires that a breaking and entering be unlawful—ie, without the owner's permission or legal privilege. A business gives the public permission to enter its premises during business hours, but not when it is closed.

A man and a woman, both purse snatchers, independently entered a grocery store, each with the intent to steal purses from unsuspecting shoppers. The man approached a victim, grabbed the strap of the purse that was hanging from her shoulder, and pulled. The victim screamed and tried to hold onto the purse, but the man knocked her down, causing her to lose her grip on the purse. The man then escaped with the purse. During the commotion that arose when the man's victim screamed, the woman approached the shopping cart of a shopper who had turned toward the noise and wandered away from her cart. The woman reached into the now-unattended shopping cart, grabbed the shopper's purse, and quickly walked away, also escaping with a purse. Both purse snatchers were apprehended a short while later. Which of the two purse snatchers could properly be convicted of robbery?

To convict a defendant of robbery, the prosecution must prove all of the following: The defendant unlawfully took and carried away the victim's property. The property was taken from the victim's person or presence by force or intimidation. The defendant intended to permanently deprive the victim of the property (ie, to steal). The force element of robbery is only met if the defendant uses more force than necessary to take the property. This means that a sudden snatching of property does not amount to robbery unless the victim resists and the defendant uses force to obtain or keep the property. Here, the man intentionally stole the victim's purse from her shoulder. Although he did not use the requisite force when he pulled on the victim's purse strap, this element was met when the victim tried to hold onto her purse and the man knocked her down to obtain it. As a result, he can be convicted of robbery

A man told his neighbor that he was going away for two weeks and asked the neighbor to keep an eye on his house. The neighbor agreed. The man gave the neighbor a key to use to check on the house. The neighbor decided to have a party in the man's house. He invited a number of friends. One friend went into the man's bedroom, took some of the man's rings, and put them in his pocket. Which of the following is true?

Trespass is the unlawful entry of a structure—ie, an entry that occurs without legal necessity or the possessor's permission. And a permissible entry can become unlawful if the entering person exceeds the scope of permission—eg, receiving permission to wait in a lobby but exploring an office. Here, the man gave the neighbor permission to enter by asking the neighbor to watch the man's house and giving the neighbor a key. And since the man did not restrict the neighbor's use of or access to the house, the neighbor did not exceed the scope of the man's permission by having a party. Therefore, the neighbor is not guilty of trespass (Choice C). Burglary is the trespassory entry of a structure with the intent to commit any crime therein. Since the neighbor's entry was permissible (not trespassory) and there is no indication that he intended to commit a crime inside, he is not guilty of burglary (Choices A & B). The friend's entry was also permissible since the neighbor (the possessor) invited him in. Therefore, the friend is not guilty of burglary (Choice A). Larceny is the (1) unlawful taking and carrying away of another's personal property with (2) the intent to permanently deprive the owner of that property. Here, the neighbor did not take any of the man's property (no larceny). But the friend took the man's rings (taking) and put them in his pocket (carrying away) to presumably keep them (intent to permanently deprive). Therefore, the friend is guilty of larceny. Educational objective:Trespass is the unlawful entry of a structure—ie, an entry without legal necessity or permission. Trespass rises to burglary if the person enters the structure with the intent to commit any crime therein. And larceny is the unlawful taking and carrying away of another's personal property with the intent to permanently deprive the owner thereof.

A woman was in the process of moving and decided to hire a mover to move the contents of her apartment across town. Upon the completion of the move, the mover sought payment from the woman for the moving services. The woman, without justification, refused to pay the mover. In a jurisdiction that has adopted the Model Penal Code with regard to theft, is the woman likely guilty of theft?

Under the Model Penal Code and state criminal codes, the common-law property offenses of larceny, embezzlement, false pretenses, and receipt of stolen goods are treated as a single statutory crime of theft. Additionally, the definition of property has been expanded from tangible personal property (the common law view) to include anything of value—eg, intangibles, services, and documents. This means that a theft conviction can stem from the unlawful taking of anything of value. Here, the woman refused to pay the mover for services it had rendered. Since the woman took something of value from the mover, she is likely guilty of theft.

A bartender asked a woman to spend a weekend with him at his apartment and promised her they would get married on the following Monday. The woman agreed and also promised the bartender that she would not tell anyone of their plans. Unknown to the woman, the bartender had no intention of marrying her. After the woman came to his apartment, the bartender told her he was going for cigarettes. He called the woman's father and told him that he had his daughter and would kill her if he did not receive $100,000. The bartender was arrested on Sunday afternoon when he went to pick up the $100,000. The woman was still at the apartment and knew nothing of the bartender's attempt to get the money. Of which crime is the bartender guilty?

Under the modern view, kidnapping is (1) the unlawful and intentional confinement of another coupled with (2) the movement or concealment of that person. Confinement occurs when the victim's freedom of movement is restricted against his/her will—ie, by force, threat of force, or deception. As a result, there is no kidnapping if the victim consents to the confinement. Here, the man convinced the woman to spend the weekend with him by falsely promising to marry her. But his false promise did not restrict the woman to the apartment against her will—she remained there voluntarily. As a result, the man cannot be convicted of kidnapping (Choices A & C). And he cannot be convicted of attempted kidnapping because he (1) never intended to kidnap the woman and (2) did not commit an overt act in furtherance of that crime. Instead, he merely intended to trick the father into paying a $100,000 ransom (Choices B & C). Educational objective:Under the modern view, kidnapping is the unlawful and intentional confinement of another that involves the movement or concealment of that person. Since confinement occurs when a person's movement is restricted against his/her will, consent is a defense to kidnapping.

A man and his friend were competitive marathon runners, and the man was determined to win a race that was upcoming in two days. The man invited his friend over for dinner, and afterward, the man asked his friend if he wanted to see the new bar that he had recently built in his basement. The friend agreed, and the man led him into his basement. When the friend entered the basement, the man locked the door behind him. The man left the friend in the basement, ran the marathon, and returned two days later to let him out. The friend immediately reported the man to the police, who subsequently arrested and charged the man with kidnapping. Is the man guilty of kidnapping?

Under the modern view, kidnapping is: the intentional* and unlawful confinement of another against that person's will (eg, by force, threat, or fraud) coupled with either moving or concealing that person. Concealment occurs when the victim is kept in a hidden location—ie, a place where the victim is not likely to be found—for a substantial period of time. Here, the friend was unlawfully confined against his will after the man lured the friend into the basement to see the new bar. The friend was then locked in the basement for two days—a substantial period of time. And since the man's basement is a place where the friend was not likely to be found, the man is guilty of kidnapping.

A woman's ex-husband was getting remarried. The woman believed that he was making a mistake and that she needed to stop him. On the day of his wedding, held outdoors at a park, the woman hid behind some bushes she knew her ex-husband would have to pass before entering the area where the ceremony would take place. When her ex-husband was a few feet away, the woman got his attention and told him to come behind the bushes to talk. When he resisted, the woman pulled out a gun and the husband reluctantly walked around the bushes to talk to her. The woman then proceeded to tell him that he could not get married. Once the ex-husband was able to convince the woman that he would not get married, he was able to leave and notified the police. The woman was subsequently arrested and charged with kidnapping. Is the woman guilty of kidnapping?

Under the modern view, kidnapping is: the intentional* and unlawful confinement of another against that person's will (eg, by force, threat, or fraud) coupled with either moving or hiding that person. Confinement occurs when the victim's freedom of movement is significantly restricted. It is not enough that the victim is prevented from taking a path or entering an area; the victim must be prevented from leaving an area or compelled to go somewhere the victim does not want to go. Additionally, the victim need only be moved a short distance to satisfy the movement requirement. Here, the ex-husband was intentionally confined when the woman threatened him with a gun to compel him to come behind the bushes against his will (Choice A). And since the man was forced to walk a few feet, the movement requirement was also satisfied (Choice B). Therefore, the woman is guilty of kidnapping.

Unprepared for a final examination, a student asked his girlfriend to set off the fire alarms in the university building 15 minutes after the test commenced. The girlfriend did so. Several students were injured in the panic that followed as people were trying to get out of the building. If both the student and the girlfriend are prosecuted for battery and for conspiracy to commit battery, what is the likely result?

Unprepared for a final examination, a student asked his girlfriend to set off the fire alarms in the university building 15 minutes after the test commenced. The girlfriend did so. Several students were injured in the panic that followed as people were trying to get out of the building. If both the student and the girlfriend are prosecuted for battery and for conspiracy to commit battery, what is the likely result? Conspiracy requires proof that (1) the defendant entered an agreement with the specific intent to commit a crime and (2) an overt act was committed in furtherance of that agreement. Here, the student and the girlfriend agreed that she would pull the fire alarms. But since they did not enter that agreement with the specific intent to commit battery, they are not guilty of conspiracy to commit battery

A state statute makes it a crime to "knowingly possess a controlled substance analogue." The statute defines a "controlled substance analogue" (CSA) as a substance that has a chemical structure similar to a controlled substance and has a physiological effect on humans similar to or greater than a controlled substance. A defendant was arrested for possession of a synthetic stimulant known by its street name as "bath salts" and was charged with violating the criminal statute. Regarding the knowledge element of this crime as it relates to the substance found in the defendant's possession, which of the following is the most proper jury instruction

When a statute makes it a crime to knowingly possess an item, the word "knowingly" applies to the statute's verb (possess) as well as the object of that verb (controlled substance analogue (CSA)). As a result, a conviction under this statute requires proof that the defendant knowingly possessed the substance and either: knew that the substance was prohibited by law—even if the defendant did not know the specific identity of the substance—or knew facts about the substance that made possessing it illegal—eg, knew that it had a chemical structure similar to a controlled substance and physiological effects on humans similar to or greater than a controlled substance—even if the defendant did not know that possessing it was illegal. A jury instruction regarding the knowledge element of this crime as it relates to the substance found in the defendant's possession must include both of the above bullet points (Choice A). The jury instruction should therefore read: "If you find that the defendant knew that the substance he possessed was illegal (prohibited by law) or had a chemical structure similar to a controlled substance and physiological effects on humans similar to or greater than a controlled substance (facts making its possession illegal), you must find the defendant guilty." (Choices C & D) Mere possession of a substance later identified as a controlled substance or an analogue thereof is not sufficient to convict the defendant. Nor is it sufficient that the substance possessed by the defendant is commonly known (by the public at large) as a CSA. Instead, the defendant must have known that the substance is illegal or known facts about the substance that made possessing it illegal. In prosecutions for possession of a controlled substance or analogue thereof, the defendant's knowledge that the substance was a controlled substance or analogue may be established by evidence that the defendant knew (1) the substance was prohibited by law or (2) facts about the substance that made possessing it illegal.

A woman was envious of her coworker for receiving a promotion they had both sought. After receiving the promotion, the coworker bought himself a very expensive watch to celebrate. The woman wanted to buy a similar watch, but she could not afford to do so. The coworker later invited the woman to attend a game night he was hosting, and the woman concocted a plan to steal the watch from him. When the woman arrived at the coworker's house the following night, the coworker invited her in and showed her to the living room where they were playing games. Later in the evening, when no one was looking, the woman snuck into the coworker's bedroom, opened the bedroom closet, and rummaged through the closet until she found a display box in which the coworker kept his valuables. As anticipated, she found the watch inside the box, hid the watch in her jacket, and left with it at the end of the night. The coworker reported the incident upon discovering that his watch was missing. The police interviewed all of the guests who attended the game night, and after seeing the woman wearing the watch in plain view, the police arrested the woman and charged her with burglary. Is the woman guilty of burglary?

When, as here, the question does not indicate that the majority rule is being tested, apply the common law rule for burglary. Common-law burglary is the: unlawful breaking and entering of another's dwelling at nighttime with the specific intent to commit a felony (eg, larceny) therein. A breaking is accomplished when even slight force is used to create an opening into a dwelling—eg, opening an unlocked door or window. If the defendant instead entered the dwelling with consent, a breaking can still occur if the defendant later enters part of the dwelling structure (eg, opening a closet door or wall safe) without permission. Here, the woman went to the coworker's house with the intent to steal his watch. But since the coworker invited the woman into his home for a game night, she did not break into his dwelling when she initially entered his home. However, she did break into the coworker's bedroom closet—part of the dwelling structure that she did not have permission to enter—later that evening. And since the woman entered the closet with the specific intent to steal the coworker's watch (ie, commit larceny), she is guilty of burglary. For burglary, a breaking can still occur after a defendant enters a dwelling with consent if the defendant subsequently enters part of the dwelling structure (eg, opening a closet door or wall safe) without consent. But merely opening an object within a dwelling (eg, desk drawer, trunk, box) is not a breaking.


संबंधित स्टडी सेट्स

PADI, Open Water Diver, Final Exam Review

View Set

1st 25 US States & Capitals (ABC order)

View Set

GMU IT 343 Midterm Chpt 1-8 review quizzes

View Set

Property and causality insurance terms and related concepts

View Set

BUSINESS CONCERNS - DEREGULATION AND BANKRUPTCYQUIZ NO. 3

View Set